Mathcenter Forum  

Go Back   Mathcenter Forum > ค้นหาในห้อง
สมัครสมาชิก คู่มือการใช้ รายชื่อสมาชิก ปฏิทิน ค้นหา ข้อความวันนี้ ทำเครื่องหมายอ่านทุกห้องแล้ว

แสดงผลลัพธ์ตั้งแต่ 1 ถึง 25 จากทั้งหมด 30
ใช้เวลาค้นหา 0.00 วินาที.
ค้นหา: ข้อความของคุณ: Anonymer
ห้อง: ปัญหาคณิตศาสตร์ทั่วไป 04 พฤษภาคม 2008, 15:28
คำตอบ: 5
เปิดอ่าน: 2,585
ข้อความของคุณ Anonymer
เป้นคล้ายๆวงล้อแล้วหมุนไปเรื่อยๆค่ะ...

เป้นคล้ายๆวงล้อแล้วหมุนไปเรื่อยๆค่ะ เดี๋ยววาดภาพออกมาให้ดูนะคะ
ห้อง: ปัญหาคณิตศาสตร์ทั่วไป 04 พฤษภาคม 2008, 15:08
คำตอบ: 47
เปิดอ่าน: 21,297
ข้อความของคุณ Anonymer
น่าจะขึ้นราคาเป็น 56.25 บาทนะคะ (ล้อเล่นน่ะค่ะ) :D

น่าจะขึ้นราคาเป็น 56.25 บาทนะคะ (ล้อเล่นน่ะค่ะ) :D
ห้อง: ปัญหาคณิตศาสตร์ทั่วไป 04 พฤษภาคม 2008, 14:57
คำตอบ: 11
เปิดอ่าน: 2,445
ข้อความของคุณ Anonymer
Fermat's Last Theorem กล่าวว่า $a^n+b^n=c^n$...

Fermat's Last Theorem กล่าวว่า
$a^n+b^n=c^n$ เมื่อ $n\in \mathbb{N} $ แล้ว $a,b,c\in \mathbb{I} $ ก็ต่อเมื่อ $n\leqslant 2$ ค่ะ :)
ห้อง: ปัญหาคณิตศาสตร์ ม. ต้น 02 พฤษภาคม 2008, 23:06
คำตอบ: 31
เปิดอ่าน: 7,568
ข้อความของคุณ Anonymer
ข้อสุดท้ายตอน 3 ว่าไงเหรอคะ

ข้อสุดท้ายตอน 3 ว่าไงเหรอคะ
ห้อง: ปัญหาคณิตศาสตร์ ม. ต้น 01 พฤษภาคม 2008, 16:40
คำตอบ: 7
เปิดอ่าน: 29,881
ข้อความของคุณ Anonymer
สมการเชิงเส้น 2...

สมการเชิงเส้น 2 ตัวแปรก็เช่น
$x+y=23,x-y=13$
ก็เอาสมการแรกบวกสมการสองก็จะได้ค่า $x=18$ ค่ะ
**สมการเชิงเส้น ดีกรีเป็น 1 เท่านั้นนะคะ
ห้อง: พีชคณิต 01 พฤษภาคม 2008, 14:18
คำตอบ: 6
เปิดอ่าน: 2,635
ข้อความของคุณ Anonymer
สุดยอดจริงๆเลยค่ะ

สุดยอดจริงๆเลยค่ะ
ห้อง: ปัญหาคณิตศาสตร์ ม.ปลาย 01 พฤษภาคม 2008, 14:08
คำตอบ: 4
เปิดอ่าน: 3,805
ข้อความของคุณ Anonymer
จะได้ว่า $x(x-3)(x-2)(x+1)\leqslant0$...

จะได้ว่า $x(x-3)(x-2)(x+1)\leqslant0$ แล้วก็พอดกราฟออกมาค่ะ จะได้ออกมาเป็นช่วง บวก กับ ช่วงลบค่ะ
ห้อง: ปัญหาคณิตศาสตร์ ม. ต้น 30 เมษายน 2008, 22:10
คำตอบ: 31
เปิดอ่าน: 7,568
ข้อความของคุณ Anonymer
เว็บนี้มัไวรัสอ่ะค่ะ Anti Virus มันขึ้นค่ะ

เว็บนี้มัไวรัสอ่ะค่ะ Anti Virus มันขึ้นค่ะ
ห้อง: ข้อสอบโอลิมปิก 30 เมษายน 2008, 01:12
คำตอบ: 30
เปิดอ่าน: 12,486
ข้อความของคุณ Anonymer
อันนี้ของ ป6 ใช่มั้ยคะ (ถ้าของ...

อันนี้ของ ป6 ใช่มั้ยคะ (ถ้าของ มัธยมนี่ง่ายมากๆๆๆเลยค่ะ) ก็สมควรที่เป็นของ ป6 แล้วล่ะค่ะ
ห้อง: ปัญหาคณิตศาสตร์ ม. ต้น 29 เมษายน 2008, 00:35
คำตอบ: 7
เปิดอ่าน: 2,226
ข้อความของคุณ Anonymer
เดี๋ยวหนูช่วยอธิบายให้ค่ะ $(\frac{a}{b}-\frac{b}{...

เดี๋ยวหนูช่วยอธิบายให้ค่ะ

$(\frac{a}{b}-\frac{b}{a})^2=1$
$\frac{a^2}{b^2}-2+\frac{b^2}{a^2}=1$
$\frac{a^2}{b^2}+2+\frac{b^2}{a^2}=5$
$(\frac{a}{b}+\frac{b}{a})^2={5}...
ห้อง: ปัญหาคณิตศาสตร์ ม. ต้น 28 เมษายน 2008, 11:50
คำตอบ: 7
เปิดอ่าน: 2,226
ข้อความของคุณ Anonymer
นำสมการแรกไปยกกำลังสอง แล้วบวก 4...

นำสมการแรกไปยกกำลังสอง แล้วบวก 4 แล้วติดรูทอีกกทีค่ะ
แล้วพอนำสมการที่ได้มาสุดท้ายมาคูณ 2 จะได้เป็นคำตอบค่ะ
ห้อง: ปัญหาคณิตศาสตร์ ม. ต้น 28 เมษายน 2008, 11:48
คำตอบ: 25
เปิดอ่าน: 5,438
ข้อความของคุณ Anonymer
IMC เหรอคะ หนูได้ ชุดแรก 16 ข้อ ชุดที่สอง...

IMC เหรอคะ หนูได้ ชุดแรก 16 ข้อ
ชุดที่สอง 11ข้อ
ชุดที่สาม 4 ข้ออ่ะค่ะ
ห้อง: ปัญหาคณิตศาสตร์ ม. ต้น 28 เมษายน 2008, 11:47
คำตอบ: 31
เปิดอ่าน: 7,568
ข้อความของคุณ Anonymer
ที่ว่า 70 เปอร์เซนได้เหรียญทองนี่คือของ กทม...

ที่ว่า 70 เปอร์เซนได้เหรียญทองนี่คือของ กทม ใช่มั้ยคะ

แล้วคนที่มีสิทธิ์ไปสอบรอบต่อไปต้องได้เหรียญทองเท่านั้นรึเปล่าคะ

ขอบคุณมากค่ะ
ห้อง: ปัญหาคณิตศาสตร์ ม. ต้น 25 เมษายน 2008, 23:36
คำตอบ: 13
เปิดอ่าน: 3,183
ข้อความของคุณ Anonymer
ข้อนี้มันจะเข้าลูปค่ะ คือ 7 9 3 1 แล้ว 5555 หาร 4...

ข้อนี้มันจะเข้าลูปค่ะ คือ 7 9 3 1 แล้ว 5555 หาร 4 เหรือเศษ 3 เพราะฉะนั้น จะได้หลักหน่วยเป็นตัวที่ 3 ก็คือ 3 อ่ะค่ะ (ถ้าเหลือเศษ 1 จะเป็นตัวที่ 1 คือ 7) ที่พูดมาได้มาจากการ Mod ค่ะ
ห้อง: ปัญหาคณิตศาสตร์ ม. ต้น 25 เมษายน 2008, 22:46
คำตอบ: 31
เปิดอ่าน: 7,568
ข้อความของคุณ Anonymer
ช่วยบอกเกณการได้เหรียญหน่อยนะคะ อยากรู้อ่ะค่ะ...

ช่วยบอกเกณการได้เหรียญหน่อยนะคะ อยากรู้อ่ะค่ะ ว่าทำได้เท่าไหร่ได้เหรียญอะไรอ่ะค่ะ ขอบคุณมากค่ะ
ห้อง: ปัญหาคณิตศาสตร์ ม. ต้น 24 เมษายน 2008, 19:30
คำตอบ: 13
เปิดอ่าน: 3,117
ข้อความของคุณ Anonymer
ข้อ 1 ตอบ 4 ค่ะ ลองใช้ Math Inductin ดูค่ะ แทน n=...

ข้อ 1 ตอบ 4 ค่ะ ลองใช้ Math Inductin ดูค่ะ แทน n= 1, 2, 3 ดูแล้วจะได้ทุกค่าเท่ากันค่ จะได้ n= 64 ก็มีค่าเป็น 4 ด้วยค่ะ
ห้อง: ปัญหาคณิตศาสตร์ ม. ต้น 24 เมษายน 2008, 19:27
คำตอบ: 3
เปิดอ่าน: 2,232
ข้อความของคุณ Anonymer
ช้อสอบ IMC ค่ะ รอบเขตปี 2551 นี่ล่ะค่ะ

ช้อสอบ IMC ค่ะ รอบเขตปี 2551 นี่ล่ะค่ะ
ห้อง: ปัญหาคณิตศาสตร์ ม. ต้น 20 เมษายน 2008, 22:10
คำตอบ: 2
เปิดอ่าน: 1,613
ข้อความของคุณ Anonymer
อ่อได้แล้วค่ะ ขอบคุณท่าน breeze123 มากๆๆเลยนะคะ...

อ่อได้แล้วค่ะ ขอบคุณท่าน breeze123 มากๆๆเลยนะคะ ตอนนี้ทำได้แล้วค่ะ
ห้อง: ปัญหาคณิตศาสตร์ ม.ปลาย 20 เมษายน 2008, 18:32
คำตอบ: 4
เปิดอ่าน: 1,412
ข้อความของคุณ Anonymer
23 ค่ะ ใช้ Mod เข้าไปไม่กี่ครั้งก็ได้แล้วค่ะ

23 ค่ะ ใช้ Mod เข้าไปไม่กี่ครั้งก็ได้แล้วค่ะ
ห้อง: ปัญหาคณิตศาสตร์ ม. ต้น 20 เมษายน 2008, 15:47
คำตอบ: 9
เปิดอ่าน: 2,498
ข้อความของคุณ Anonymer
เอาโจทย์ให่ค่ะ ก็เป็นโจทย์ปานกลางค่ะ...

เอาโจทย์ให่ค่ะ ก็เป็นโจทย์ปานกลางค่ะ แต่ก็น่าสนใจดีนะคะ

1. จงแยกตัวประกอบของ $$(a+b+c)^4-(a+b)^4-(b+c)^4-(c+a)^4+a^4+b^4+c^4$$

**ข้อนี้ไม่จำเป็นต้องกระจายค่ะมองดีๆค่ะ ลองแทน $P(a,b,c)=>...
ห้อง: ซอฟต์แวร์คณิตศาสตร์ 20 เมษายน 2008, 15:41
คำตอบ: 1
เปิดอ่าน: 6,284
ข้อความของคุณ Anonymer
อยากได้โปรแกรม Maple อ่ะค่ะ

ใครพอมีช่วยอัพลงเว็บช่วยโหลดหรือช่วยบอกลิ้งค์โหลดด้วยนะคะ ขอบคุณมาเลยค่ะ
ห้อง: ปัญหาคณิตศาสตร์ ม. ต้น 19 เมษายน 2008, 20:18
คำตอบ: 2
เปิดอ่าน: 1,613
ข้อความของคุณ Anonymer
ขอถามโจทย์เรขาหน่อยค่ะ ยากมากๆๆเลยอ่ะค่ะ

1.) สามเหลี่ยมหน้าจั่ว มีด้านประกอบมุมยอดเป็น 3 เท่าของฐาน ซึ่งมีทั้งหมด 8 ณุปเท่ากันทุกประการนำมาประกอบกันเป็นรูป 6 เหลี่ยม PQRSTU แล้ว จงหาค่าของ $RU^2+QT^2+3PS$

2.) สามเหลี่ยม$PQR$ มี $A,B,C$...
ห้อง: ปัญหาคณิตศาสตร์ ม. ต้น 18 เมษายน 2008, 20:33
คำตอบ: 18
เปิดอ่าน: 6,686
ข้อความของคุณ Anonymer
ข้อนี้เป็นไงอ่ะค่ะ ไม่แน่ใจนะคะ หนูคิดว่า...

ข้อนี้เป็นไงอ่ะค่ะ ไม่แน่ใจนะคะ หนูคิดว่า ข้อนี้อาจจะต้องใช้ Mod เข้าช่วยอ่ะค่ะ แต่หนูยังไม่ออกเลยค่ะ

ส่วนเว็บที่ท่าน Kanakon ให้มามันเปิดไม่ได้นะคะ เดี๋ยวเปิดได้เมื่อไหร่หนูจะมาเปิดค่ะ ขอบคุณมากค่ะ
ห้อง: ปัญหาคณิตศาสตร์ ม. ต้น 18 เมษายน 2008, 20:30
คำตอบ: 18
เปิดอ่าน: 4,602
ข้อความของคุณ Anonymer
ก็ถ้าเกิดไม่เท่ามาแล้ว...

ก็ถ้าเกิดไม่เท่ามาแล้ว มันจะเหลือพจน์กลางออกมาเต็มเลยอ่ะค่ะ ซึ่งมันก็ผิดแน่นอนน่ะค่ะ (เหตุผลที่ง่ายที่สุดค่ะ) :happy:
ห้อง: ปัญหาคณิตศาสตร์ ม. ต้น 15 เมษายน 2008, 17:23
คำตอบ: 18
เปิดอ่าน: 4,602
ข้อความของคุณ Anonymer
ข้อนี้หนูได้แล้วล่ะค่ะ...

ข้อนี้หนูได้แล้วล่ะค่ะ ขอบคุณทุกท่านที่มาช่วยตอบนะคะ

คือวิธีที่หนูคิดคือแบบนี้ค่ะ

$$\displaystyle{(\sum_{i = 1}^{n}a_i)(\sum_{i=1}^{n}a_i^3)}=96\times 216=144^2$$
$$\therefore...
แสดงผลลัพธ์ตั้งแต่ 1 ถึง 25 จากทั้งหมด 30

 
ทางลัดสู่ห้อง

เวลาที่แสดงทั้งหมด เป็นเวลาที่ประเทศไทย (GMT +7) ขณะนี้เป็นเวลา 17:23


Powered by vBulletin® Copyright ©2000 - 2024, Jelsoft Enterprises Ltd.
Modified by Jetsada Karnpracha